06.09.2021 Views

Linear Algebra, Theory And Applications, 2012a

Linear Algebra, Theory And Applications, 2012a

Linear Algebra, Theory And Applications, 2012a

SHOW MORE
SHOW LESS

Create successful ePaper yourself

Turn your PDF publications into a flip-book with our unique Google optimized e-Paper software.

274 LINEAR TRANSFORMATIONS CANONICAL FORMS<br />

4. Find the minimal polynomial for<br />

⎛<br />

A = ⎝<br />

1 2 3<br />

2 1 4<br />

−3 2 1<br />

by the above technique assuming the field of scalars is the rational numbers. Is what<br />

you found also the characteristic polynomial?<br />

5. Show, using the rational root theorem, the minimal polynomial for A in the above<br />

problem is irreducible with respect to Q. Letting the field of scalars be Q find the<br />

rational canonical form and a similarity transformation which will produce it.<br />

6. Find the rational canonical form for the matrix<br />

⎛<br />

1 2 1 −1<br />

⎜ 2 3 0 2<br />

⎝ 1 3 2 4<br />

1 2 1 2<br />

7. Let A : Q 3 → Q 3 be linear. Suppose the minimal polynomial is (λ − 2) ( λ 2 +2λ +7 ) .<br />

Find the rational canonical form. Can you give generalizations of this rather simple<br />

problem to other situations?<br />

8. Find the rational canonical form with respect to the field of scalars equal to Q for the<br />

matrix<br />

⎛<br />

0 0 1<br />

⎞<br />

A = ⎝ 1 0 −1 ⎠<br />

0 1 1<br />

Observe that this particular matrix is already a companion matrix of λ 3 − λ 2 + λ − 1.<br />

Then find the rational canonical form if the field of scalars equals C or Q + iQ.<br />

9. Let q (λ) be a polynomial and C its companion matrix. Show the characteristic and<br />

minimal polynomial of C are the same and both equal q (λ).<br />

10. ↑Use the existence of the rational canonical form to give a proof of the Cayley Hamilton<br />

theorem valid for any field, even fields like the integers mod p for p a prime. The earlier<br />

proof based on determinants was fine for fields like Q or R where you could let λ →∞<br />

but it is not clear the same result holds in general.<br />

11. Suppose you have two n×n matrices A, B whose entries are in a field F and suppose G<br />

is an extension of F. For example, you could have F = Q and G = C. Suppose A and<br />

B are similar with respect to the field G. Can it be concluded that they are similar<br />

with respect to the field F? Hint: First show that the two have the same minimal<br />

polynomial over F. Next consider the proof of Lemma 10.8.3 and show that they have<br />

the same rational canonical form with respect to F.<br />

⎞<br />

⎠<br />

⎞<br />

⎟<br />

Hooray! Your file is uploaded and ready to be published.

Saved successfully!

Ooh no, something went wrong!